Question

A circular thin wire loop of radius a=37.2 cm and total resistance R=0.186 12 is positioned in the (xy)-plane, where it is ex

0 0
Add a comment Improve this question Transcribed image text
Answer #1

E Criven a= 37.2cm -01872m az I Resistance FR = 0.1862 2 Bent (A) = Bordt + + Bt2 sy Z where Bo = 6.685 d=60072 7 T/s 8 = 690(2) Due to this in the loop. induced Emf, there will be cussent flow nal ( 2- 28t) R [ cerere R I E By this this Induced curr

Add a comment
Know the answer?
Add Answer to:
A circular thin wire loop of radius a=37.2 cm and total resistance R=0.186 12 is positioned...
Your Answer:

Post as a guest

Your Name:

What's your source?

Earn Coins

Coins can be redeemed for fabulous gifts.

Not the answer you're looking for? Ask your own homework help question. Our experts will answer your question WITHIN MINUTES for Free.
Similar Homework Help Questions
  • Electricity and Magnetism: Will rate high for correct solutions! A circular thin wire loop of radius...

    Electricity and Magnetism: Will rate high for correct solutions! A circular thin wire loop of radius a=37.6 cm and total resistance R=0.188 12 is positioned in the (xy)-plane, where it is exposed to the external (applied) uniform magnetic field directed along the z- axis. Over some period of time t, the z-component of that field varies as Bextz(t)=Bo – at + ßt?, where Bo=1.852 T, a=9.8 T/s and B=6.481 T/s2. Right at the geometrical center of the loop circle, there...

  • Q1. (25 points) A circular loop of wire of resistance R = 4 N and radius...

    Q1. (25 points) A circular loop of wire of resistance R = 4 N and radius r = 30 cm is in a uniform magnetic field directed out of the page as shown in the figure. a) (7 points) If a clockwise current of 1 = 20 mA is induced in the loop, is the magnetic field increasing or decreasing in time? b) (8 points) Find the induced emf in the loop. c) (10 points) Find the rate at which...

  • A circular loop of wire of resistance - 0.500 and radius 8.10 cm is in a...

    A circular loop of wire of resistance - 0.500 and radius 8.10 cm is in a uniform magnetic field directed out of the page as in the figure below. A clockwise current of 120 mA is induced in the loop. (a) is the magnetic field increasing or decreasing in time? decreasing (b) Find the rate at which the field is changing with time.

  • (11%) Problem 8: A circular wire loop of radius r = 0.35 m and resistance R=...

    (11%) Problem 8: A circular wire loop of radius r = 0.35 m and resistance R= 11 12 rotates about a shaft through its diameter at a constant rate of f= 5.5 Hz in a uniform B = 0.21-T magnetic field directed perpendicular to the rotation axis. The plane of the loop is perpendicular to the magnetic field at time t = 0. 4 17% Part (a) Select the correct expression for the time-dependent magnetic flux through the loop. 17%...

  • A circular loop of wire of resistance 0.4 Ohm and radius 11 cm, sitting n the...

    A circular loop of wire of resistance 0.4 Ohm and radius 11 cm, sitting n the plane of this paper, is in a uniform magnetic field, directed out of the paper. A counterclockwise induced current of 3.5 mA is measured in the loop. Is the magnetic field increasing or decreasing as time elapses? Explain how you know. At what rate is the magnetic field changing with time?

  • Problem 1 (20 points] A circular loop of wire with radius r = 10 cm and...

    Problem 1 (20 points] A circular loop of wire with radius r = 10 cm and Resistance R = 1 N is * in a region of uniform magnetic field, as shown in the figure. The magnetic field is directed into the plane. At t = 0s, the magnetic field * * is zero. Then, the magnetic field starts to increase as function of time, B(t) = 0.5t? * * * X X a) [5 points) is the magnetic flux...

  • Q1. (25 points) A circular loop of wire of resistance R = 1 N and radius...

    Q1. (25 points) A circular loop of wire of resistance R = 1 N and radius r = 15 cm is in a uniform magnetic field directed out of the page as shown in the figure. a) (7 points) If a clockwise current of I = 8 mA is induced in the loop, is the magnetic field increasing or decreasing in time? b) (8 points) Find the induced emf in the loop. c) (10 points) Find the rate at which...

  • os A circular loop of wire of radius 1.55cm is in a uniform magnetic field, with...

    os A circular loop of wire of radius 1.55cm is in a uniform magnetic field, with the plane of the loop perpendicular to the direction of the field. The magnetic field varies with time according to B(t) = 0.064 + 1.2t, where t is in seconds, and B is in T. Calculate the magnetic flux through the loop at t0 s. B Submit Answer Tries 0/10 Calculate the magnitude of the emf induced in the loop. Submit Answer Tries 0/10...

  • A circular loop of wire of radius 7.24 cm is in a uniform magnetic field, with...

    A circular loop of wire of radius 7.24 cm is in a uniform magnetic field, with the plane of the loop perpendicular to the direction of the field. The magnetic field varies with time according to B(t) = 0.014 + 1.1t, where t is in seconds, and B is in T. Calculate the magnetic flux through the loop at t = 0 s. B

  • A circular loop of wire of radius 11.5 cm is placed in a magnetic field directed...

    A circular loop of wire of radius 11.5 cm is placed in a magnetic field directed perpendicular to the plane of the loop as shown in the figure below. If the field decreases at the rate of 0.043 0 T/s in some time interval, find the magnitude of the emf induced in the loop during this interval.

ADVERTISEMENT
Free Homework Help App
Download From Google Play
Scan Your Homework
to Get Instant Free Answers
Need Online Homework Help?
Ask a Question
Get Answers For Free
Most questions answered within 3 hours.
ADVERTISEMENT
ADVERTISEMENT
ADVERTISEMENT